This is the first RMO Mock test from MOMC Season 2. All the credit and authorship of the problems belongs to their respective sources.
I will be discussing the solutions of the problems along with how I motivated them. So if you are planning to attempt the mock, beware of the spoilers ahead.
Problem 1 (Canada 1993) Show that the number is rational if and only if three distinct terms that form a geometric progression can be chosen from the sequence
Solution: One direction of the problem is much easier than the other. If there exist three distinct terms such that then can be shown to be equal to . Therefore is rational.
Let where are integers and . For the converse, consider the following triple:
It is easy to verify that this triple works. The triple can be found by considering a general triple and then working out the divisibility constraints.
However, we are not done yet! We need to ensure that each term of the triple is . Thus our proof fails if . In fact, it fails for as well, as the terms of the triple won't be distinct. For we can just pick . For negative the trick is to see that we can just consider a large positive integer such that . Then letting and picking our tuple as before works.
Problem 2 (239 2014 S6) Given positive real numbers such that . Prove that .
Solution: The difficult observation is that the infinite sum:
is equal to . This suggests that all equal to is a pseudo equality case. Therefore, we are motivated to apply AM-GM Inequality to and :
(Had we taken and , the exponents would not be nice after AM-GM). This implies that . Summing this inequality over gives:
Problem 3 (HMMT 2023 Feb Teams) Let be a convex quadrilateral such that for some angle . Point lies inside the quadrilateral such that . Prove that .
Solution: The official source provides 4 amazing solutions to the problem here. My solution was the same as the first one.
Problem 4 (All Russian Olympiad 2000) Let and . Prove that if , then
Solution: This is a non standard inequality as all regular methods fail. The key observation is that is invariant when we shift the sequence by any constant. Further observe that is always if . It would be easy to show if each of the individual summands are negative. For that we need that is positive. But we can ensure that by shifting the sequence such that and have the same sign for all . Thus we are done!
Problem 5 (HMMT 2023 Feb Teams) Let be a triangle. Point lies on segment such that . Point lies on the opposite side of line as and satisfies and . Analogously, point lies on the opposite side of line as and satisfies and . Prove that lines and are perpendicular.
Solution: The official source provides beautiful proofs to this superb problem here
Problem 6 (239 2017 J1) Let be a permutation of . Prove that the sum
taken over all permutations equals .
Solution: We show the result by induction on . The base case is clear. We would, in fact, show the general result for a permutation of any set of real numbers (The reason for this would be soon clear). For any , consider all the permutations in which appears last. By using the induction hypothesis, the corresponding sum would be equal to . Note that we used the general form of the result here. Hence the total sum is:
which completes the induction hypothesis.
Side note: I had not anticipated the huge workload that is associated with the college admissions process. So I was not able to put out any mocks in the past two months. Nonetheless, I plan to post 1-2 more mocks in the coming days. Have a look out for those!